Difference between revisions of "2001 AIME II Problems/Problem 3"

 
m
Line 1: Line 1:
 
== Problem ==
 
== Problem ==
 +
Given that
 +
<center><math>\begin{eqnarray*}x_{1}&=&211,\\ x_{2}&=&375,\\ x_{3}&=&420,\\ x_{4}&=&523, \textrm{ and}\\ x_{n}&=&x_{n-1}-x_{n-2}+x_{n-3}-x_{n-4}\textrm{ when }n\geq5, \end{eqnarray*}</math></center>
 +
find the value of <math>x_{531}+x_{753}+x_{975}</math>.
  
 
== Solution ==
 
== Solution ==
 +
{{solution}}
  
 
== See also ==
 
== See also ==
* [[2001 AIME II Problems]]
+
{{AIME box|year=2001|n=II|num-b=2|num-a=4}}

Revision as of 00:40, 20 November 2007

Problem

Given that

$\begin{eqnarray*}x_{1}&=&211,\\ x_{2}&=&375,\\ x_{3}&=&420,\\ x_{4}&=&523, \textrm{ and}\\ x_{n}&=&x_{n-1}-x_{n-2}+x_{n-3}-x_{n-4}\textrm{ when }n\geq5, \end{eqnarray*}$ (Error compiling LaTeX. Unknown error_msg)

find the value of $x_{531}+x_{753}+x_{975}$.

Solution

This problem needs a solution. If you have a solution for it, please help us out by adding it.

See also

2001 AIME II (ProblemsAnswer KeyResources)
Preceded by
Problem 2
Followed by
Problem 4
1 2 3 4 5 6 7 8 9 10 11 12 13 14 15
All AIME Problems and Solutions